Đến nội dung

Hình ảnh

Topic tổng hợp một số bất đẳng thức trong kì thi MO các nước

* * * * * 16 Bình chọn

  • Please log in to reply
Chủ đề này có 501 trả lời

#441
dangkhuong

dangkhuong

    Sĩ quan

  • Thành viên
  • 312 Bài viết

Sau đây là lời giải cho bài 160 thực ra bài này chẳng có gì hoàn toàn lưu ý 2 đẳng thức sau là đc:

1)$\sum MA^2=8R^2$

2)$\sum MA^4=16R^4$

Từ đó sử dụng Cauchy-Schawz là đc rồi ,xong.


:ukliam2:  :ukliam2:  :ukliam2:


#442
dangkhuong

dangkhuong

    Sĩ quan

  • Thành viên
  • 312 Bài viết

Bài 161(Croatia TST): Cho a,b,c >0. CMR:

$\sqrt{\frac{a^3+c(a^2+bc)}{a(b+2c)}}+\sqrt{\frac{b^3+a(b^2+ca)}{b(c+2a)}}+\sqrt{\frac{c^3+b(c^2+ab)}{c(a+2b)}}\geq 3\sqrt{\frac{3}{a+b+c}}$


:ukliam2:  :ukliam2:  :ukliam2:


#443
nangcuong8e

nangcuong8e

    Trung sĩ

  • Thành viên
  • 134 Bài viết

Bài 158(MOSP Shortlist): Cho a,b,c>0. CMR: nếu a+b+c=3 thì

$\sum \frac{a}{a^3-a^2+1}\leq 3$

Ta có: $\frac{a}{a^3 -a^2+1} \leq 1$ tương đương với $a^3-a^2-a+1 \geq 0$

$\Leftrightarrow (a-1)^2(a+1) \geq 0$ (luôn đúng với $a>0$)

 Do đó $\frac{a}{a^3 -a^2+1} \leq 1$ đúng

$\Rightarrow \sum \frac{a}{a^3-a^2+1} \geq 1+1+1 =3$



#444
dangkhuong

dangkhuong

    Sĩ quan

  • Thành viên
  • 312 Bài viết

Bài 156(Belarus TST) : cho đường tròn tâm O và 1 điểm A nằm ngoài dường tròn. Kẻ các tiếp tuyến AB và AC đến đường tròn, kẻ cát tuyến ADE đến đường tròn, BC giao DE tại I. CMR:

$\frac{ED}{ID}+\frac{IA}{EA}\geq 2\sqrt{2}$

Dấu bằng xảy ra không? Nếu có thì khi nào? 

Sau đây là lời giải cho bài toán trên như sau, sau khi chúng ta sử dụng trực tiếp bđt AM-GM cho VT ta quy bài toán về cm hệ thức hình học đơn giản sau:

$\frac{ED}{EA}.\frac{ID}{IA}=2$

Chú ý bài toán quen thuộc sau: Cho điểm A nằm ngoài (O), kẻ các tiếp tuyến AB,AC đến (O). Kẻ cát tuyến ADE đến (O), kẻ đường thẳng qua D song song với AB cắt BC,BE tại M,N. Khi đó M là trung điểm DN. Từ đây theo định lý Thales ta có hệ thức trên. Chú ý rằng đẳng thức ko xảy ra ta thu đc đpcm.

Nhận xét: Bài này thực sự là một bài toán thuần tuý hình học viết dưới dạng đại số.


Bài viết đã được chỉnh sửa nội dung bởi dangkhuong: 30-06-2015 - 15:47

:ukliam2:  :ukliam2:  :ukliam2:


#445
dangkhuong

dangkhuong

    Sĩ quan

  • Thành viên
  • 312 Bài viết

Bài 162(Bristist MO): Cho a,b,c>0. CMR: nếu $a+b+c=ab+bc+ca$ thì

$\frac{a^2+b(b^2+c^2)}{(c+2a)^2}+\frac{b^2+c(c^2+a^2)}{(a+2b)^2}+\frac{c^2+a(a^2+b^2)}{(b+2c)^2}\geq 1$

 


Bài viết đã được chỉnh sửa nội dung bởi dangkhuong: 30-06-2015 - 15:48

:ukliam2:  :ukliam2:  :ukliam2:


#446
an1712

an1712

    Trung sĩ

  • Thành viên
  • 149 Bài viết

Bài 159(Balkan Shortlist): Cho a,b,c >0. CMR:

$\sum \frac{a^2}{a^2+ab+b^2}\geq 1$

đặt $\frac{b}{a}=x $ $\frac{c}{b}=y $ $\frac{a}{c}=z $ $\Leftrightarrow  xyz=1$

bdt$\Leftrightarrow \sum \frac{1}{1+x+x^2}\geq 1$

đặt$x=\frac{y_{1}z_{1}}{x_{1}^2}$.........

bđt $\Leftrightarrow \sum \frac{x_{1}^4}{x_{1}^4+x_{1}^2y_{1}z_{1}+y_{1}^2z_{1}^2}\geq 1$

$\Leftrightarrow \sum x_{1}^2y_{1}z_{1}\leq \sum x_{1}^2y_{1}^2$ luôn đúng


Bài viết đã được chỉnh sửa nội dung bởi an1712: 30-06-2015 - 16:00

tiến tới thành công  :D


#447
dangkhuong

dangkhuong

    Sĩ quan

  • Thành viên
  • 312 Bài viết

Bài 163(Spain TST): Cho a,b,c>0: $a^2+b^2+c^2=3$. CMR:

$\frac{a}{a^2+b^3+c^2}+\frac{b}{b^2+c^3+a^2}+\frac{c}{c^2+a^3+b^2}\leq 1$


:ukliam2:  :ukliam2:  :ukliam2:


#448
dangkhuong

dangkhuong

    Sĩ quan

  • Thành viên
  • 312 Bài viết

Bài 164(tự sáng tác dựa vào bài 161): Cho a,b,c>0. Tìm hằng số k tốt nhất để bất đẳng thức sau đúng:

$\sqrt{\frac{a^3+c(a^2+bc)}{a(b+kc)}}+\sqrt{\frac{b^3+a(b^2+ca)}{b(c+ka)}}+\sqrt{\frac{c^3+b(c^2+ab)}{c(a+kb)}}\geq 3\sqrt{\frac{3}{a+b+c}}$


:ukliam2:  :ukliam2:  :ukliam2:


#449
dangkhuong

dangkhuong

    Sĩ quan

  • Thành viên
  • 312 Bài viết

Bài 162(Bristist MO): Cho a,b,c>0. CMR: nếu $a+b+c=ab+bc+ca$ thì

$\frac{a^2+b(b^2+c^2)}{(c+2a)^2}+\frac{b^2+c(c^2+a^2)}{(a+2b)^2}+\frac{c^2+a(a^2+b^2)}{(b+2c)^2}\geq 1$

 

Sau đây là lời giải cho bài số 162, bài toán có ý tưởng hoàn toàn dựa vào bất đẳng thức Cauchy-Schawz.

Ta sử dụng đánh giá sau bằng Cauchy-Schawz:

$(a^2+b(b^2+c^2))(a^2+b+b)\geq (a^2+b^2+bc)^2\geq (2ab+bc)^2=b^2(c+2a)^2 \Leftrightarrow \frac{a^2+b(b^2+c^2)}{(c+2a)^2}\geq \frac{b^2}{a^2+2b}$

Rất tương tự ta có VT bất đẳng thức lớn hơn hoặc bằng$\sum \frac{b^2}{(a^2+2b)}$

Đến đây ta sử dụng bất đẳng thức Cauchy-Schawz và giả thuyết ta thu đc đpcm.


:ukliam2:  :ukliam2:  :ukliam2:


#450
the man

the man

    Thiếu úy

  • Thành viên
  • 589 Bài viết

Bài 163(Spain TST): Cho a,b,c>0: $a^2+b^2+c^2=3$. CMR:

$\frac{a}{a^2+b^3+c^2}+\frac{b}{b^2+c^3+a^2}+\frac{c}{c^2+a^3+b^2}\leq 1$

Sử dụng bất đẳng thức Holder:

    $(a^2+b^3+c^2)(a^2+b^3+c^2)(a^2+1+c^2)\geq (a^2+b^2+c^2)^3=27$

    $\rightarrow a^2+b^3+c^2\geq \frac{3\sqrt{3}}{\sqrt{a^2+c^2+1}}\rightarrow \frac{a}{a^2+b^3+c^2}\leq \frac{a\sqrt{a^2+c^2+1}}{3\sqrt{3}}$

    $\rightarrow \sum \frac{a}{a^2+b^3+c^2}\leq \frac{\sum a\sqrt{a^2+c^2+1}}{3\sqrt{3}}$

Sử dụng bất đẳng thức  Cauchy-Schwarz:

    $\sum a\sqrt{a^2+c^2+1}\leq \sqrt{(a^2+b^2+c^2)(2a^2+2b^2+2c^2+3)}=3\sqrt{3}$

Từ đó ta có điều phải chứng minh


"God made the integers, all else is the work of man."

                                                Leopold Kronecker


#451
the man

the man

    Thiếu úy

  • Thành viên
  • 589 Bài viết

Bài 165 (Mediterranean Mathematical Competition 2009).Chứng minh rằng với mọi  $a,b,c $ dương ta luôn có:

$$\sum \frac{ab}{a^2+ab+b^2}\leq \frac{a}{2a+b}+\frac{b}{2b+c}+\frac{c}{2c+a}$$


  • 128 yêu thích

"God made the integers, all else is the work of man."

                                                Leopold Kronecker


#452
khanghaxuan

khanghaxuan

    Trung úy

  • Thành viên
  • 969 Bài viết

Topic vắng như chùa bà đanh :( 

Bài 166 (2008 Tournament of towns) : Cho $a_{1},a_{2},...,a_{n}$ là $n$ số thực dương thỏa mãn : $\sum_{i=1}^{n}\leq \frac{1}{2}$

Chứng minh rằng : $\prod_{i=1}^{n}(1+a_{i})<2$


Điều tôi muốn biết trước tiên không phải là bạn đã thất bại ra sao mà là bạn đã chấp nhận nó như thế nào .

- A.Lincoln -

#453
tuananh2000

tuananh2000

    Thượng sĩ

  • Thành viên
  • 218 Bài viết

Bài 165 (Mediterranean Mathematical Competition 2009).Chứng minh rằng với mọi  $a,b,c $ dương ta luôn có:

$$\sum \frac{ab}{a^2+ab+b^2}\leq \frac{a}{2a+b}+\frac{b}{2b+c}+\frac{c}{2c+a}$$

Đặt $(a;b;c)=(\frac{x}{y};\frac{y}{z};\frac{z}{x})$ , đưa bđt cần cm thành

$\sum \frac{x^{2}}{x^{3}+x^{2}+1}\leq\sum \frac{x}{2x+1}$

Hay $\sum(x-1)^{2}(x+1)x\geq 0$  (đúng)

Vậy bất đẳng thức đc cm

Dấu đẳng thức xảy ra khi $a=b=c=1$ 


  • 128 yêu thích

Live more - Be more  


#454
the man

the man

    Thiếu úy

  • Thành viên
  • 589 Bài viết

Bài 153: (Việt Nam TST 2005) 

Cho $a,b,c>0$. Chứng minh: $\left ( \frac{a}{a+b} \right )^3+\left ( \frac{b}{b+c} \right )^3+\left ( \frac{c}{c+a} \right )^3 \geq \frac{3}{8}$

$P=\left ( \frac{a}{a+b} \right )^3+\left ( \frac{b}{b+c} \right )^3+\left ( \frac{c}{c+a} \right )^3=\frac{1}{\left ( 1+\frac{b}{a} \right )^3}+\frac{1}{\left ( 1+\frac{c}{b} \right )^3}+\frac{1}{\left ( 1+\frac{a}{c} \right )^3}$

    $=\frac{1}{(1+x)^3}+\frac{1}{(1+y)^3}+\frac{1}{(1+z)^3}$       Với  $\frac{b}{a}=x,\frac{c}{b}=y,\frac{a}{c}=z\rightarrow xyz=1$

Sử dụng bất đẳng thức Am-GM:

    $\frac{1}{(1+x)^3}+\frac{1}{(1+x)^3}+\frac{1}{8}\geq \frac{3}{2}.\frac{1}{(1+x)^2}\rightarrow \frac{1}{(1+x)^3}\geq \frac{3}{4}.\frac{1}{(1+x)^2}-\frac{1}{16}$

    $\rightarrow P\geq \frac{3}{4}.\left [ \frac{1}{(1+x)^2}+\frac{1}{(1+y)^2}+ \frac{1}{(1+z)^2}\right ]-\frac{3}{16}$

Ta cần chứng minh :  

            $\frac{1}{(1+x)^2}+\frac{1}{(1+y)^2}+ \frac{1}{(1+z)^2}\geq \frac{3}{4}$    (*)

Ta có bài toán quen thuộc:

Với  $m,n,p,q$  là các số dương có tích bằng 1 thì  

            $\frac{1}{(1+m)^2}+ \frac{1}{(1+n)^2}+\frac{1}{(1+p)^2}+ \frac{1}{(1+q)^2}\geq 1$

Áp dụng vào bài toán với  $m=x,n=y,p=z,q=1$ ta có  (*)

Vậy bài toán được chứng minh xong


"God made the integers, all else is the work of man."

                                                Leopold Kronecker


#455
the man

the man

    Thiếu úy

  • Thành viên
  • 589 Bài viết

Đặt $(a;b;c)=(\frac{x}{y};\frac{y}{z};\frac{z}{x})$ , đưa bđt cần cm thành

$\sum \frac{x^{2}}{x^{3}+x^{2}+1}\leq\sum \frac{x}{2x+1}$

Hay $\sum(x-1)^{2}(x+1)x\geq 0$  (đúng)

Vậy bất đẳng thức đc cm

Dấu đẳng thức xảy ra khi $a=b=c=1$ 

Thế thì $abc=1$ à


"God made the integers, all else is the work of man."

                                                Leopold Kronecker


#456
dogsteven

dogsteven

    Đại úy

  • Thành viên
  • 1567 Bài viết

$P=\left ( \frac{a}{a+b} \right )^3+\left ( \frac{b}{b+c} \right )^3+\left ( \frac{c}{c+a} \right )^3=\frac{1}{\left ( 1+\frac{b}{a} \right )^3}+\frac{1}{\left ( 1+\frac{c}{b} \right )^3}+\frac{1}{\left ( 1+\frac{a}{c} \right )^3}$

    $=\frac{1}{(1+x)^3}+\frac{1}{(1+y)^3}+\frac{1}{(1+z)^3}$       Với  $\frac{b}{a}=x,\frac{c}{b}=y,\frac{a}{c}=z\rightarrow xyz=1$

Sử dụng bất đẳng thức Am-GM:

    $\frac{1}{(1+x)^3}+\frac{1}{(1+x)^3}+\frac{1}{8}\geq \frac{3}{2}.\frac{1}{(1+x)^2}\rightarrow \frac{1}{(1+x)^3}\geq \frac{3}{4}.\frac{1}{(1+x)^2}-\frac{1}{16}$

    $\rightarrow P\geq \frac{3}{4}.\left [ \frac{1}{(1+x)^2}+\frac{1}{(1+y)^2}+ \frac{1}{(1+z)^2}\right ]-\frac{3}{16}$

Ta cần chứng minh :  

            $\frac{1}{(1+x)^2}+\frac{1}{(1+y)^2}+ \frac{1}{(1+z)^2}\geq \frac{3}{4}$    (*)

Ta có bài toán quen thuộc:

Với  $m,n,p,q$  là các số dương có tích bằng 1 thì  

            $\frac{1}{(1+m)^2}+ \frac{1}{(1+n)^2}+\frac{1}{(1+p)^2}+ \frac{1}{(1+q)^2}\geq 1$

Áp dụng vào bài toán với  $m=x,n=y,p=z,q=1$ ta có  (*)

Vậy bài toán được chứng minh xong

Áp dụng: $\dfrac{1}{(x+1)^2}+\dfrac{1}{(y+1)^2}\geqslant \dfrac{1}{xy+1}$


Quyết tâm off dài dài cày hình, số, tổ, rời rạc.


#457
tuananh2000

tuananh2000

    Thượng sĩ

  • Thành viên
  • 218 Bài viết

Thế thì $abc=1$ à

Mình cũng chẳng biết @@


Live more - Be more  


#458
Hoang Long Le

Hoang Long Le

    Thượng sĩ

  • Thành viên
  • 265 Bài viết

Thế thì $abc=1$ à

 Bất đẳng thức đồng bậc nên có thể chuẩn hoá $abc=1$

 

 

$P=\left ( \frac{a}{a+b} \right )^3+\left ( \frac{b}{b+c} \right )^3+\left ( \frac{c}{c+a} \right )^3=\frac{1}{\left ( 1+\frac{b}{a} \right )^3}+\frac{1}{\left ( 1+\frac{c}{b} \right )^3}+\frac{1}{\left ( 1+\frac{a}{c} \right )^3}$

    $=\frac{1}{(1+x)^3}+\frac{1}{(1+y)^3}+\frac{1}{(1+z)^3}$       Với  $\frac{b}{a}=x,\frac{c}{b}=y,\frac{a}{c}=z\rightarrow xyz=1$

Sử dụng bất đẳng thức Am-GM:

    $\frac{1}{(1+x)^3}+\frac{1}{(1+x)^3}+\frac{1}{8}\geq \frac{3}{2}.\frac{1}{(1+x)^2}\rightarrow \frac{1}{(1+x)^3}\geq \frac{3}{4}.\frac{1}{(1+x)^2}-\frac{1}{16}$

    $\rightarrow P\geq \frac{3}{4}.\left [ \frac{1}{(1+x)^2}+\frac{1}{(1+y)^2}+ \frac{1}{(1+z)^2}\right ]-\frac{3}{16}$

Ta cần chứng minh :  

            $\frac{1}{(1+x)^2}+\frac{1}{(1+y)^2}+ \frac{1}{(1+z)^2}\geq \frac{3}{4}$    (*)

Ta có bài toán quen thuộc:

Với  $m,n,p,q$  là các số dương có tích bằng 1 thì  

            $\frac{1}{(1+m)^2}+ \frac{1}{(1+n)^2}+\frac{1}{(1+p)^2}+ \frac{1}{(1+q)^2}\geq 1$

Áp dụng vào bài toán với  $m=x,n=y,p=z,q=1$ ta có  (*)

Vậy bài toán được chứng minh xong

 Áp dụng Holder có :

  $3\left [ \sum \left ( \frac{a}{a+b} \right )^3 \right ]^2\geq \left [ \sum \left ( \frac{a}{a+b} \right )^2 \right ]^3$

  Nên ta chỉ cần chứng minh $\sum \left ( \frac{a}{a+b} \right )^2\geq \frac{3}{4}\Leftrightarrow \sum \frac{1}{(1+x)^2}\geq \frac{3}{4}$

 Với $x=\frac{b}{a};y=\frac{a}{c};z=\frac{c}{b};xyz=1$

 Lại có :  $\frac{1}{(1+x)^2}+\frac{1}{(1+y)^2}\geq \frac{1}{(1+xy)\left ( 1+\frac{x}{y} \right )}+\frac{1}{(1+xy)\left ( 1+\frac{y}{x} \right )}$

               $=\frac{y}{(1+xy)(x+y)}+\frac{x}{(1+xy)(x+y)}=\frac{1}{1+xy}=\frac{z}{z+1}$

 Nên chỉ cần chứng minh $\frac{z}{z+1}+\frac{1}{(z+1)^2}\geq \frac{3}{4}\Leftrightarrow (z-1)^2\geq 0$

 Dấu "=" xảy ra khi $x=y=z=1\Leftrightarrow a=b=c$


Bài viết đã được chỉnh sửa nội dung bởi Hoang Long Le: 02-07-2015 - 14:55

IM LẶNG

#459
dangkhuong

dangkhuong

    Sĩ quan

  • Thành viên
  • 312 Bài viết

Bài 167(China TST): Cho a,b,c thuộc [0;1]. Tìm GTNN và GTLN của biểu thức sau:

                              P=$\frac{1}{(b-1)^2+a^2}+\frac{1}{(b-1)^2+c^2}$


Bài viết đã được chỉnh sửa nội dung bởi dangkhuong: 02-07-2015 - 15:31

:ukliam2:  :ukliam2:  :ukliam2:


#460
dogsteven

dogsteven

    Đại úy

  • Thành viên
  • 1567 Bài viết

Bài 165 (Mediterranean Mathematical Competition 2009).Chứng minh rằng với mọi  $a,b,c $ dương ta luôn có:

$$\sum \frac{ab}{a^2+ab+b^2}\leq \frac{a}{2a+b}+\frac{b}{2b+c}+\frac{c}{2c+a}$$

Áp dụng bất đẳng thức Cauchy-Schwarz:

$3-\sum \dfrac{2b}{2b+a}=\sum \dfrac{a}{a+2b}\geqslant \dfrac{(a+b+c)^2}{a^2+b^2+c^2+2ab+2bc+2ca}=1\Rightarrow \sum \dfrac{b}{a+2b}\leqslant 1$

Do đó $\sum \dfrac{a}{2a+b}=\sum \dfrac{a}{2a+b}+\sum \dfrac{b}{2b+a}-1$

Ta sẽ chứng minh: $\dfrac{a}{2a+b}+\dfrac{b}{2b+a}\geqslant \dfrac{ab}{a^2+b^2+ab}+\dfrac{1}{3}$ hay $\dfrac{a^2+b^2+4ab}{(a+2b)(b+2a)}\geqslant \dfrac{a^2+b^2+4ab}{3(a^2+b^2+ab)}\Leftrightarrow (a-b)^2\geqslant 0$


Quyết tâm off dài dài cày hình, số, tổ, rời rạc.





0 người đang xem chủ đề

0 thành viên, 0 khách, 0 thành viên ẩn danh